Difference between revisions of "Aufgaben:Exercise 3.5Z: Integration of Dirac Functions"

From LNTwww
m (Text replacement - "Category:Exercises for Signal Representation" to "Category:Signal Representation: Exercises")
Line 4: Line 4:
  
 
[[File:P_ID515__Sig_Z_3_5_neu.png|right|frame|Integration von Diracfunktionen ]]
 
[[File:P_ID515__Sig_Z_3_5_neu.png|right|frame|Integration von Diracfunktionen ]]
Like in  [[Aufgaben:3.5_Differentiation_eines_Dreicksignals|Task 3.5]]  the spectrum  ${Y(f)}$  of the signal
+
Like in  [[Aufgaben:3.5_Differentiation_eines_Dreicksignals|Exercise 3.5]]  the spectrum  ${Y(f)}$  of the signal
:$$y( t ) = \left\{ \begin{array}{c} A \\  - A \\  0 \\  \end{array} \right.\quad \begin{array}{*{20}c}  {{\rm{f \ddot{u}r}}}  \\  {{\rm{f\ddot{u} r}}}  \\  {\rm{else.}}  \\ \end{array}\;\begin{array}{*{20}c}  { - T \le t < 0,}  \\  {0 < t \le T,}  \\  {}  \\\end{array}$$
+
:$$y( t ) = \left\{ \begin{array}{c} A \\  - A \\  0 \\  \end{array} \right.\quad \begin{array}{*{20}c}  {{\rm{for}}}  \\  {{\rm{for}}}  \\  {\rm{else.}}  \\ \end{array}\;\begin{array}{*{20}c}  { - T \le t < 0,}  \\  {0 < t \le T,}  \\  {}  \\\end{array}$$
can be determined. Again,&nbsp; $A = 1 \,\text{V}$&nbsp; and&nbsp; $T = 0.5 \,\text{ms}$ apply.
+
can be determined.&nbsp; Again,&nbsp; $A = 1 \,\text{V}$&nbsp; and&nbsp; $T = 0.5 \,\text{ms}$&nbsp; apply.
  
 
Assume the time signal&nbsp; ${x(t)}$&nbsp; according to the middle sketch, which is composed of three Dirac pulses at&nbsp; $–T$,&nbsp; $0$&nbsp; and&nbsp; $+T$&nbsp; with the pulse weights&nbsp; ${AT}$,&nbsp; $-2{AT}$&nbsp; und&nbsp; ${AT}$&nbsp;.
 
Assume the time signal&nbsp; ${x(t)}$&nbsp; according to the middle sketch, which is composed of three Dirac pulses at&nbsp; $–T$,&nbsp; $0$&nbsp; and&nbsp; $+T$&nbsp; with the pulse weights&nbsp; ${AT}$,&nbsp; $-2{AT}$&nbsp; und&nbsp; ${AT}$&nbsp;.
  
The spectral function&nbsp; ${X(f)}$&nbsp; can be given directly by applying the&nbsp; [[Signal_Representation/Fourier_Transform_Laws#Vertauschungssatz|Vertauschungssatz]]&nbsp; if one takes into account that the time function belonging to&nbsp; ${U(f)}$&nbsp; is as follows (see lower sketch):
+
The spectral function&nbsp; ${X(f)}$&nbsp; can be given directly by applying the&nbsp; [[Signal_Representation/Fourier_Transform_Theorems#Duality_Theorem|Duality Theorem]]&nbsp; if one takes into account that the time function belonging to&nbsp; ${U(f)}$&nbsp; is as follows (see lower sketch):
 
:$$u( t ) =  - 2A + 2A \cdot \cos ( {2{\rm{\pi }}f_0 t} ).$$
 
:$$u( t ) =  - 2A + 2A \cdot \cos ( {2{\rm{\pi }}f_0 t} ).$$
 
 
  
  
Line 21: Line 19:
  
 
''Hints:''  
 
''Hints:''  
*This exercise belongs to the chapter&nbsp; [[Signal_Representation/Fourier_Transform_Laws|Fourier Transform Laws]].
+
*This exercise belongs to the chapter&nbsp; [[Signal_Representation/Fourier_Transform_Theorems|Fourier Transform Theorems]].
*All of these laws are illustrated with examples in the learning video&nbsp; [[Gesetzmäßigkeiten_der_Fouriertransformation_(Lernvideo)|Gesetzmäßigkeiten der Fouriertransformation]]&nbsp;.
+
*All the laws presented here  are illustrated with examples in the (German language) learning video<br> &nbsp; &nbsp; &nbsp;[[Gesetzmäßigkeiten_der_Fouriertransformation_(Lernvideo)|Gesetzmäßigkeiten der Fouriertransformation]] &nbsp; &rArr; &nbsp;  "Regularities to the Fourier transform".
The following relationship exists between&nbsp; ${x(t)}$&nbsp; and&nbsp; ${y(t)}$&nbsp;:
+
*The following relationship exists between&nbsp; ${x(t)}$&nbsp; and&nbsp; ${y(t)}$&nbsp;:
 
:$$y( t ) = \frac{1}{T} \cdot \hspace{-0.1cm} \int_{ - \infty }^{\hspace{0.05cm}t} {x( \tau  )}\, {\rm d}\tau .$$
 
:$$y( t ) = \frac{1}{T} \cdot \hspace{-0.1cm} \int_{ - \infty }^{\hspace{0.05cm}t} {x( \tau  )}\, {\rm d}\tau .$$
*The&nbsp; [[Signal_Representation/Fourier_Transform_Laws#Integration_Theorem|Integration_Theorem]]&nbsp; reads in a correspondingly adapted form:
+
*The&nbsp; [[Signal_Representation/Fourier_Transform_Laws#Integration_Theorem|Integration Theorem]]&nbsp; reads in a correspondingly adapted form:
 
:$$\frac{1}{T}\cdot \hspace{-0.1cm} \int_{ - \infty }^{\hspace{0.05cm}t} {x( \tau  )}\,\, {\rm d}\tau\ \ \circ\!\!-\!\!\!-\!\!\!-\!\!\bullet\, \ \ X( f ) \cdot \left( {\frac{1}{{{\rm{j}}\cdot 2{\rm{\pi }\cdot }fT}} + \frac{1}{2T}\cdot {\rm \delta} ( f )} \right).$$
 
:$$\frac{1}{T}\cdot \hspace{-0.1cm} \int_{ - \infty }^{\hspace{0.05cm}t} {x( \tau  )}\,\, {\rm d}\tau\ \ \circ\!\!-\!\!\!-\!\!\!-\!\!\bullet\, \ \ X( f ) \cdot \left( {\frac{1}{{{\rm{j}}\cdot 2{\rm{\pi }\cdot }fT}} + \frac{1}{2T}\cdot {\rm \delta} ( f )} \right).$$
 
   
 
   
Line 33: Line 31:
  
 
<quiz display=simple>
 
<quiz display=simple>
{Calculate the spectral function&nbsp; ${X(f)}$. What is its magnitude at the frequencies&nbsp; $f = 0$&nbsp; and&nbsp; $f = 1\, \text{kHz}$?
+
{Calculate the spectral function&nbsp; ${X(f)}$.&nbsp; What is its magnitude at the frequencies&nbsp; $f = 0$&nbsp; and&nbsp; $f = 1\, \text{kHz}$?
 
|type="{}"}
 
|type="{}"}
 
$|{X(f = 0)}| \ = \ $ { 0. }  &nbsp;$\text{mV/Hz}$
 
$|{X(f = 0)}| \ = \ $ { 0. }  &nbsp;$\text{mV/Hz}$
Line 39: Line 37:
  
  
{Calculate the spectral function&nbsp; ${Y(f)}$. What values result at the frequencies&nbsp; $f = 0$&nbsp; and&nbsp; $f = 1\, \text{kHz}$?
+
{Calculate the spectral function&nbsp; ${Y(f)}$.&nbsp; What values result at the frequencies&nbsp; $f = 0$&nbsp; and&nbsp; $f = 1\, \text{kHz}$?
 
|type="{}"}
 
|type="{}"}
 
$|{Y(f = 0)}|\ = \ $ { 0. }  &nbsp;$\text{mV/Hz}$
 
$|{Y(f = 0)}|\ = \ $ { 0. }  &nbsp;$\text{mV/Hz}$

Revision as of 14:40, 27 April 2021

Integration von Diracfunktionen

Like in  Exercise 3.5  the spectrum  ${Y(f)}$  of the signal

$$y( t ) = \left\{ \begin{array}{c} A \\ - A \\ 0 \\ \end{array} \right.\quad \begin{array}{*{20}c} {{\rm{for}}} \\ {{\rm{for}}} \\ {\rm{else.}} \\ \end{array}\;\begin{array}{*{20}c} { - T \le t < 0,} \\ {0 < t \le T,} \\ {} \\\end{array}$$

can be determined.  Again,  $A = 1 \,\text{V}$  and  $T = 0.5 \,\text{ms}$  apply.

Assume the time signal  ${x(t)}$  according to the middle sketch, which is composed of three Dirac pulses at  $–T$,  $0$  and  $+T$  with the pulse weights  ${AT}$,  $-2{AT}$  und  ${AT}$ .

The spectral function  ${X(f)}$  can be given directly by applying the  Duality Theorem  if one takes into account that the time function belonging to  ${U(f)}$  is as follows (see lower sketch):

$$u( t ) = - 2A + 2A \cdot \cos ( {2{\rm{\pi }}f_0 t} ).$$




Hints:

$$y( t ) = \frac{1}{T} \cdot \hspace{-0.1cm} \int_{ - \infty }^{\hspace{0.05cm}t} {x( \tau )}\, {\rm d}\tau .$$
$$\frac{1}{T}\cdot \hspace{-0.1cm} \int_{ - \infty }^{\hspace{0.05cm}t} {x( \tau )}\,\, {\rm d}\tau\ \ \circ\!\!-\!\!\!-\!\!\!-\!\!\bullet\, \ \ X( f ) \cdot \left( {\frac{1}{{{\rm{j}}\cdot 2{\rm{\pi }\cdot }fT}} + \frac{1}{2T}\cdot {\rm \delta} ( f )} \right).$$


Questions

1

Calculate the spectral function  ${X(f)}$.  What is its magnitude at the frequencies  $f = 0$  and  $f = 1\, \text{kHz}$?

$|{X(f = 0)}| \ = \ $

 $\text{mV/Hz}$
$|{X(f = 1\, \text{kHz})}|\ = \ $

 $\text{mV/Hz}$

2

Calculate the spectral function  ${Y(f)}$.  What values result at the frequencies  $f = 0$  and  $f = 1\, \text{kHz}$?

$|{Y(f = 0)}|\ = \ $

 $\text{mV/Hz}$
$|{Y(f = 1\, \text{kHz})}| \ = \ $

 $\text{mV/Hz}$


Solution

(1)  In the description of the task you will find the Fourier correspondence between  ${u(t)}$  and  ${U(f)}$.

  • Since both the time functions  ${u(t)}$  and  ${x(t)}$  and the corresponding spectra  ${U(f)}$  and  ${X(f)}$  are even and real,  ${X(f)}$  can be easily calculated by applying the "Vertauschungssatz":
$$X( f ) = - 2 \cdot A \cdot T + 2 \cdot A \cdot T \cdot \cos \left( {{\rm{2\pi }}fT} \right).$$
  • Because of the relation  $\sin^{2}(\alpha) = (1 – \cos(\alpha))/2$  it can also be written for this:
$$X( f ) = - 4 \cdot A \cdot T \cdot \sin ^2 ( {{\rm{\pi }}fT} ).$$
  • At frequency  $f = 0$   ${x(t)}$  has no spectral components  ⇒   ${X(f = 0)} \;\underline{= 0}$.
  • For  $f = 1 \,\text{kHz}$  – also  $f \cdot T = 0.5$  –   on the other hand:
$$X( f = 1\;{\rm{kHz}} ) = - 4 \cdot A \cdot T = -2 \cdot 10^{ - 3} \;{\rm{V/Hz}}\; \Rightarrow \; |X( {f = 1\;{\rm{kHz}}} )| \hspace{0.15 cm}\underline{= 2 \;{\rm{mV/Hz}}}{\rm{.}}$$


(2)  The spectrum  ${Y(f)}$  can be determined from  ${X(f)}$  by applying the integration theorem.

  • Because of  ${X(f = 0)} = 0$  the Dirac function does not have to be taken into account at the frequency  $f = 0$  and one obtains:
$$Y( f ) = \frac{X( f )}{{{\rm{j}} \cdot 2{\rm{\pi }}fT}} = \frac{{ - 4 \cdot A \cdot T \cdot \sin ^2 ( {{\rm{\pi }}fT} )}}{{{\rm{j}}\cdot 2{\rm{\pi }}fT}} = 2{\rm{j}} \cdot A \cdot T \cdot \frac{{\sin ^2 ( {{\rm{\pi }}fT} )}}{{{\rm{\pi }}fT}}.$$
  • Of course, the result is the same as in  Task 3.5:
  • At frequency  $f = 0$   ${y(t)}$  also has no spectral components  ⇒   ${Y(f = 0)} \;\underline{= 0}$.
  • For  $f = 1\,\text{kHz} \ \ (f \cdot T = 0.5)$  one obtains a value smaller by a factor  $\pi$  compared to  $X(f)$ :
$$|Y( {f = 1\;{\rm{kHz}}} )| = \frac{4 \cdot A \cdot T}{\rm{\pi }} \hspace{0.15 cm}\underline{= {\rm{0}}{\rm{.636}} \;{\rm{mV/Hz}}}{\rm{.}}$$